1
$\begingroup$

My question is regarding the LR cost function from andrews ML course (http://feature-space.com/en/document50.pdf , page -5)

$cost= \frac{1}{m}[ -y \times \log(\psi) - (1-y) \times \log(\kappa) ]$

The vector y holds values for the digits (1-10), so if we plug these values in the cost function then the cost function takes ambiguous values. For instance if y=5, then the cost function will have both the parameters

$cost (y=4) = \frac{1}{m} [ -4 \times \log(\psi) -(1+4) \times \log(\kappa) ]$

As per Andrew's lecture I remember him saying that only one of the log terms would remain inside the cost function as if classified correct $\log(\psi)$ term remains else $\log(\kappa)$ remains.

Please help me where I'm getting it wrong.

$\endgroup$
1
  • 2
    $\begingroup$ Remember that y is a vector in this case, not a scalar. $\endgroup$
    – Bar
    Jun 10, 2015 at 13:21

1 Answer 1

1
$\begingroup$

$y$ always takes on values of 1 or 0, as you noted. For the multi-class problem, you're going to solve for the "one vs. all" case. You'll need to transform your $y$ vector into a vector of 1's and 0's depending on the class you are minimizing for. So for the number 5, you'll solve for $P(y=5)$ vs. $P(y \ne 5)$. You repeat that for all your digits. You come up with 10 different $h_{i}{\theta}$'s, i.e. $h_{1i}{\theta}$, $h_{2i}{\theta}$, ...

Also, the following is from your link at the bottom of page 8:

When training the classifier for class k ∈ {1, ..., K}, you will want a m-dimensional vector of labels y, where yj ∈ 0, 1 indicates whether the j-th training instance belongs to class k (yj = 1), or if it belongs to a different class (yj = 0).

$\endgroup$

Your Answer

By clicking “Post Your Answer”, you agree to our terms of service and acknowledge you have read our privacy policy.

Not the answer you're looking for? Browse other questions tagged or ask your own question.